Violympic toán 9

Nguyễn Thu Trà

Cho x, y, z > 0 thoả mãn: \(xy+yz+zx=3xyz\). Chứng minh rằng: \(\frac{x^3}{z+x^2}+\frac{y^3}{x+y^2}+\frac{z^3}{y+z^2}\ge\frac{1}{2}\left(\frac{1}{x}+\frac{1}{y}+\frac{1}{z}\right)\)

Akai Haruma
25 tháng 5 2019 lúc 21:41

Lời giải:
Áp dụng BĐT AM-GM ta có:

\(\text{VT}=x-\frac{x}{x^2+z}+y-\frac{y}{y^2+x}+z-\frac{z}{z^2+y}=(x+y+z)-\left(\frac{x}{x^2+z}+\frac{y}{y^2+x}+\frac{z}{z^2+y}\right)\)

\(\geq (x+y+z)-\left(\frac{x}{2\sqrt{x^2z}}+\frac{y}{2\sqrt{y^2x}}+\frac{z}{2\sqrt{z^2y}}\right)=(x+y+z)-\frac{1}{2}\left(\frac{1}{\sqrt{x}}+\frac{1}{\sqrt{y}}+\frac{1}{\sqrt{z}}\right)(1)\)

Từ giả thiết \(xy+yz+xz=3xyz\Rightarrow \frac{1}{x}+\frac{1}{y}+\frac{1}{z}=3\)

Cauchy-Schwarz:

\(3=\frac{1}{x}+\frac{1}{y}+\frac{1}{z}\geq \frac{9}{x+y+z}\Rightarrow x+y+z\geq 3(2)\)

\(\left(\frac{1}{\sqrt{x}}+\frac{1}{\sqrt{y}}+\frac{1}{\sqrt{z}}\right)^2\leq (\frac{1}{x}+\frac{1}{y}+\frac{1}{z})(1+1+1)=9\)

\(\Rightarrow \left(\frac{1}{\sqrt{x}}+\frac{1}{\sqrt{y}}+\frac{1}{\sqrt{z}}\right)\leq 3(3)\)

Từ \((1);(2);(3)\Rightarrow \text{VT}\geq 3-\frac{1}{2}.3=\frac{3}{2}\)

Mặt khác: \(\text{VP}=\frac{1}{2}(\frac{1}{x}+\frac{1}{y}+\frac{1}{z})=\frac{3}{2}\)

Do đó \(\text{VT}\geq \text{VP}\) (đpcm)

Dấu "=" xảy ra khi $x=y=z=1$


Bình luận (0)

Các câu hỏi tương tự
Khởi My
Xem chi tiết
Văn Thắng Hồ
Xem chi tiết
Hoàng Quốc Tuấn
Xem chi tiết
Nguyễn Bùi Đại Hiệp
Xem chi tiết
Clgt
Xem chi tiết
Kakarot Songoku
Xem chi tiết
Hàn Thiên Băng
Xem chi tiết
fghj
Xem chi tiết
Lê Đình Quân
Xem chi tiết